An interesting integral with the floor function.

We present an interesting integral involving the floor function.
Playlist: • Interesting Integrals
www.michael-penn.net
www.researchgate.net/profile/...
www.randolphcollege.edu/mathem...

Пікірлер: 169

  • @TheOneSevenNine
    @TheOneSevenNine4 жыл бұрын

    I've never seen anyone write a summation sign with such ferocity

  • @blackpenredpen
    @blackpenredpen4 жыл бұрын

    You opened my eyes to new and crazy things!! And that camera angle switch at 6:55 was fresh!

  • @MichaelPennMath

    @MichaelPennMath

    4 жыл бұрын

    Thanks, I found another tripod at the thrift store a few months ago and finally put it to good use!

  • @paillote

    @paillote

    4 жыл бұрын

    This channel should be called "theFloorFuncionGuy"

  • @FireStormOOO_

    @FireStormOOO_

    3 жыл бұрын

    Indeed. Kinda surprising calculus is helpful on something so agresssively discontinuous.

  • @gradecracker
    @gradecracker4 жыл бұрын

    Its crazy that the steps you used were so bizarre and unconventional but it still got you to the same answer! Absolutely brilliant!

  • @ireallydontknow3299
    @ireallydontknow32994 жыл бұрын

    That was cool. I'm happy that I was able to solve this on my own, but only because I was able to leave the answer in function of the sum S = 1 - 1/2 + 1/3 - 1/4 + ..., which I happened to know was ln(2).

  • @alessandropizzotti932

    @alessandropizzotti932

    4 жыл бұрын

    Yes, that is a much easier way.

  • @virajagr

    @virajagr

    3 жыл бұрын

    Yes, from the Taylor series of ln(x+1)

  • @shrirammaiya9867

    @shrirammaiya9867

    2 жыл бұрын

    @@virajagr no directly from the integral

  • @lyrimetacurl0

    @lyrimetacurl0

    2 жыл бұрын

    For me I was like "I wonder if it's going to be ln(2)" without really knowing why I thought it 😂

  • @technoguyx
    @technoguyx4 жыл бұрын

    Love how simple the final expression is. Btw the audio seems to clip a lot more than in previous videos.

  • @FadkinsDiet

    @FadkinsDiet

    3 жыл бұрын

    I'd guess the recording level was too low and he had to up the gain in post

  • @Nomnomlick
    @Nomnomlick4 жыл бұрын

    I noticed very quickly that the integral from 1/n to 1/n+1 was just 1/n-1/n+1 with alternating signs starting with negative. So you get -(1-1/2) + (1/2-1/3) - (1/3-1/4) + ... Getting rid of the paranthesis and grouping terms with common denominator you get: -1 + 2/2 - 2/3 + 2/4 - 2/5. Substracting and adding 1, factoring out -2 you get 1-2AlternateHarmonic. Found ln2 by finding that the Maclaurin series of ln(1+x) fit the alternating harmonic series pretty well.

  • @abushahid1150
    @abushahid11504 жыл бұрын

    Also if you remember the Taylor series of ln(1+x) and vigilant enough, you could easy avoid like last 5 steps.

  • @1089S
    @1089S Жыл бұрын

    You make it sound like a fine poetry and that is awesome!

  • @PatrickOfTav
    @PatrickOfTav2 жыл бұрын

    "And that finishes this video". Have I got the right Michael Penn?

  • @el_variable
    @el_variable4 жыл бұрын

    I love the way you make your videos. Keep it up, I learned a lot from you :)

  • @adamjennifer6437

    @adamjennifer6437

    3 жыл бұрын

    Just watch this impressive Math channel kzread.info/dron/ZDkxpcvd-T1uR65Feuj5Yg.html

  • @peterdecupis8296
    @peterdecupis82962 жыл бұрын

    Very cool work! As a matter of fact, the function is well integrable in Peano sense, because the discontinuity of the floor function occurs only in a numerable set of isolated points; then we can correctly replace the improper integral with a series whose terms are definite integrals relevant to adjacent unit intervals [k, k+1), for any positive index k. Then the final evaluation of the series is correctly performed by exploiting the convergence condition of alternate sign series with vanishing terms; a quicker approach to the final evaluation is based on the Taylor-McLaurin series expansion of the function f(x)=ln(1+x) for x=1, i.e. f(1)=ln2, that is immediately recognized as the alternate harmonic series.

  • @wannabeactuary01
    @wannabeactuary013 жыл бұрын

    Love this and the use towards the end of the anti-derivative is icing on a cake!

  • @willyh.r.1216
    @willyh.r.12164 жыл бұрын

    This is amazingly mind blowing stuff, man! Encore, encore, encore...please.

  • @norajcarnaj9207

    @norajcarnaj9207

    2 жыл бұрын

    pk tu fais le français là mon reuf?

  • @phee4174
    @phee41742 жыл бұрын

    another nice property of this integral is that if you replace 1/x with x^s you get something that's essentially the alternating Riemann zeta function (though it's evaluated at something like 1/s instead of s, and is multiplied by and has added to it a factor of 1/x^2)

  • @stasiawright377
    @stasiawright3774 жыл бұрын

    I've never seen these tricks before, thank you!

  • @suniltshegaonkar7809
    @suniltshegaonkar78093 жыл бұрын

    Great Michael, it is a very nice problem, never seen any problem close to this.

  • @fivestar5855
    @fivestar58552 жыл бұрын

    That's brilliant one! Especially with geometric series representation.

  • @Salvador964
    @Salvador964 Жыл бұрын

    Gracias por tan interesante ejercicio.

  • @spideramazon5032
    @spideramazon50324 жыл бұрын

    Very nice integral evaluation. i love watching your videos a lot.

  • @adamjennifer6437

    @adamjennifer6437

    3 жыл бұрын

    Just watch this impressive Math channel kzread.info/dron/ZDkxpcvd-T1uR65Feuj5Yg.html

  • @mathunt1130
    @mathunt11303 жыл бұрын

    The substitution was key here. It made things very clear to see and is clearly a very good technique for these types of problems. Finding the series by integration was also a neat trick.

  • @minwithoutintroduction
    @minwithoutintroduction Жыл бұрын

    رائع جدا كالعادة. أحب هذه التكاملات الغريبة

  • @douglasmagowan2709
    @douglasmagowan27094 жыл бұрын

    By minute 7, we had an expression that was obvious to anyone who knew the Taylor series of ln (1+x). But rather than saying that, or deriving the Taylor series, the expanation becomes circuitious over its last 5 minutes.

  • @zwz.zdenek

    @zwz.zdenek

    4 жыл бұрын

    It's about teaching, not about finishing first.

  • @user-fh5rm2ef4n
    @user-fh5rm2ef4n4 жыл бұрын

    Hey, interesting problem. Thanks. You've got a tiny mistake 2:55. You've forgotten du in your substitution.

  • @adamjennifer6437

    @adamjennifer6437

    3 жыл бұрын

    Just watch this impressive Math channel kzread.info/dron/ZDkxpcvd-T1uR65Feuj5Yg.html

  • @alexpavlov6754
    @alexpavlov67543 жыл бұрын

    I think You are using famous math sum, but you resolve sum in the great wonderful manner. Thank you

  • @AltinoSantos
    @AltinoSantos4 жыл бұрын

    Very interesting. The function graph would also be welcome. Congratulations on the channel.

  • @adamjennifer6437

    @adamjennifer6437

    3 жыл бұрын

    Just watch this impressive Math channel kzread.info/dron/ZDkxpcvd-T1uR65Feuj5Yg.html

  • @wolfmanjacksaid
    @wolfmanjacksaid4 жыл бұрын

    I love the deadpan "great", keep it up!

  • @ByteOfCake

    @ByteOfCake

    3 жыл бұрын

    He's just containing all his excitement inside of him :3

  • @Walczyk
    @Walczyk3 жыл бұрын

    awesome problem!

  • @erickherrerapena8981
    @erickherrerapena89814 жыл бұрын

    Increíble resolución.

  • @JonathonV
    @JonathonV4 жыл бұрын

    That was incredible! Very cool integral and algebra/calculus substitutions. I also really liked your camera angle switch at 6:55 so I could see the board. You should keep the camera there! And for some more unsolicited but hopefully still welcome advice: I suggest moving your “please like and subscribe” thing to the end of the video so that your promo thumbnails for other videos don't block your grand reveal! I had to scrub back and forth through the video so I could see the solution on the board. Anyway, cool problem!

  • @adamjennifer6437

    @adamjennifer6437

    3 жыл бұрын

    Just watch this impressive Math channel kzread.info/dron/ZDkxpcvd-T1uR65Feuj5Yg.html

  • @nailabenali7488
    @nailabenali74884 жыл бұрын

    I've just seen this on my book this morning and yet here I am watching your video because it's much more interesting, maybe if I start working on summable families you may do a video about it too xD! Thank you a lot !!!

  • @MichaelPennMath

    @MichaelPennMath

    4 жыл бұрын

    I think this is a "famous" problem. I have seen it before, but I don't really know where.

  • @user-yt8xc8zw6v

    @user-yt8xc8zw6v

    4 жыл бұрын

    @@MichaelPennMath This is work of Dr Ovidiu Furdui , you can find it in his research papers or even in his book entitled by (Limits, Series, and Fractional Part Integrals)

  • @Someone-cr8cj
    @Someone-cr8cj3 жыл бұрын

    THAT'S SO GREAT!

  • @TheMichaelmorad
    @TheMichaelmorad Жыл бұрын

    You could just use a little bit of reasoning: Between 1/(n+1) and 1/n the function evaluates at (-1)ⁿ which means that this expression is the sun where 1

  • @zapahaha
    @zapahaha3 жыл бұрын

    amazing!!!

  • @mryip06
    @mryip062 жыл бұрын

    amazing

  • @pikkutonttu2697
    @pikkutonttu26974 жыл бұрын

    I was about to try the substitition method, but I backed off as the wikipedia article on integration by substitution gives a theorem, which assumes that the integrand is a continuous function on the interval of integration. I suppose the assumption can be relieved at least to piecewise continuity, since your initial and subsequent methods give the same result.

  • @laventin4332
    @laventin43324 жыл бұрын

    Hey can you make a video on the 4th isomorphism theorem and composition series. Thanks a ton !

  • @mushroomsteve
    @mushroomsteve4 жыл бұрын

    I got stuck at 10:55 when you said "this is absolutely convergent". It looks like the series in question is just a different form of sum(0,inf,(-1)^n/(n+1)), which is an alternating harmonic series, and thus not absolutely convergent.

  • @tracyh5751

    @tracyh5751

    4 жыл бұрын

    You are right. Absolute convergence does not work here. Instead, he can use the dominated convergence theorem along with the alternating series partial sum estimate to justify the interchange.

  • @mushroomsteve

    @mushroomsteve

    4 жыл бұрын

    @@tracyh5751 Thank you! I was really confused about that step.

  • @jordiplotnikovpous4844

    @jordiplotnikovpous4844

    4 жыл бұрын

    Tracy H which equals 1/2 right?

  • @Hiltok

    @Hiltok

    4 жыл бұрын

    Sum = 1 - 1/2 + 1/3 + 1/4 - 1/5 ... = [1 + 1/2] + [1/3 - 1/4] + ... # this is the 'alternating partial sum' Tracy H mentions = Sum 1/(n^2+n) for n=1,2,3,... < Sum 1/n^2 = finite (= pi^2/6) # this is the dominated convergence

  • @alephnull4044

    @alephnull4044

    4 жыл бұрын

    @@Hiltok Very nice. Although you wouldn't need sum 1/n^2 = pi^2/6, just sum 1/(n^2+n) = 1.

  • @lason91
    @lason913 жыл бұрын

    awesome

  • @ningliu3648
    @ningliu36483 жыл бұрын

    hi Penn, I got confused about some of the steps. Since you only have the property of conditional convergence, you are not allowed to change the order of summation to guarantee you reach the same number. What you did in your video, you change the order of the summation.

  • @dansheppard2965

    @dansheppard2965

    2 жыл бұрын

    He did say something to justify this, something about the harmonic series, but I didn't catch it.

  • @feitao13

    @feitao13

    Жыл бұрын

    @@dansheppard2965 He said that it is absolutely convergent, which is not true.

  • @cicik57
    @cicik57 Жыл бұрын

    okay, you can move easier and straightvorward way. Basic idea with whole part or fraction part is to split integral on parts where this prackets give the same value. for example, you can split intevral 0..1 on : I(1/2 ..1) of -1 dx + I(1/3 ..1/2) of 1 dx + I(1/4 ..1/3) of -1 dx +...= -x evaluated in (1, 1/2) +x | (1/2..1/3) - x(1/3, 1/4)... = -1 + 1/2 + 1/2 -1/3-1/3 + 1/4 + 1/4 = the sign change harmonic series is well known and is ln2, so -1 + 2(1-ln2) = 1-2ln2

  • @operationmike8553
    @operationmike85534 жыл бұрын

    You are brilliant.

  • @adamjennifer6437

    @adamjennifer6437

    3 жыл бұрын

    Just watch this impressive Math channel kzread.info/dron/ZDkxpcvd-T1uR65Feuj5Yg.html

  • @antoniopalacios8160
    @antoniopalacios81604 жыл бұрын

    I get -1+2ln2 for the case of the ceiling function instead of the floor function. Thank you.

  • @anshusingh1493
    @anshusingh14934 жыл бұрын

    Thanks for giving me new insight...........its brilliant...hve subscribed ur channel...

  • @adamjennifer6437

    @adamjennifer6437

    3 жыл бұрын

    Just watch this impressive Math channel kzread.info/dron/ZDkxpcvd-T1uR65Feuj5Yg.html

  • @br75857
    @br758574 жыл бұрын

    Dude you are awesome

  • @vbcool83
    @vbcool834 жыл бұрын

    Got the answer as 1-2ln(2) - let's see if it turns out right! Very strange that this crazy power integral involving floors related to logs!

  • @danielhan2007

    @danielhan2007

    4 жыл бұрын

    because you can solve this problem geometrically, i.e., alternating sums of area of rectangles with lengths of (1-1/2), (1/2-1/3), ... and height of 1 and -1.

  • @adamjennifer6437

    @adamjennifer6437

    3 жыл бұрын

    Just watch this impressive Math channel kzread.info/dron/ZDkxpcvd-T1uR65Feuj5Yg.html

  • @trevorsong4345
    @trevorsong43454 жыл бұрын

    Amazing!!!

  • @TheFinav
    @TheFinav4 жыл бұрын

    Great.

  • @filipsperl
    @filipsperl2 жыл бұрын

    this is so cool

  • @JoJoModding
    @JoJoModding4 жыл бұрын

    10:58 "because this is absolutely convergent" Is it? If you consider the absolute sum you just get 1/n which diverges.

  • @MichaelPennMath

    @MichaelPennMath

    4 жыл бұрын

    That’s my bad, it’s actually because of the dominated convergence theorem.

  • @harlock7521

    @harlock7521

    4 жыл бұрын

    @@MichaelPennMath because of alternating series test also

  • @martinepstein9826

    @martinepstein9826

    3 жыл бұрын

    @@harlock7521 That's just a convergence test. It doesn't tell you whether you can interchange the integral and the sum.

  • @liyi-hua2111
    @liyi-hua21113 жыл бұрын

    (i didn't learn really well at math which sadly was my major.) would you mind to tell me: 1. how can "sum((-1)^n*(1/n-1/n+1)),n=1 to inf" split into two parts? from what i recall, alternating series can converge to different result by proper rearrangement. Won't splitting things up make a different result? 2. Considering "integral (-1)^floor(1/x), from 0 to 1" as an alternating series formed by area pieces, integration seems to be no order to converge to a certain answer, or are there any conventions or theorems for this kind of integration? it would be grateful if you could reply to my questions.

  • @Camll
    @Camll Жыл бұрын

    Loveeeed❤❤❤❤❤

  • @hypernova4334
    @hypernova43343 жыл бұрын

    How does the geometric series converge at the right endpoint of the interval x=1? Shouldn’t |x| < 1?

  • @cycklist
    @cycklist4 жыл бұрын

    Fancy new camera angle 👌

  • @MichaelPennMath

    @MichaelPennMath

    4 жыл бұрын

    Thanks!

  • @lesprivatrizal
    @lesprivatrizal4 жыл бұрын

    Great

  • @bautibunge737
    @bautibunge7372 жыл бұрын

    Can you actually separate the sum if the series doesn't converge absolutely?

  • @zackmercurys
    @zackmercurys4 жыл бұрын

    I think your mic is clipping for some reason. might require a replacement

  • @davidblauyoutube
    @davidblauyoutube Жыл бұрын

    The correct answer, 1 - 2 ln 2, is negative because the integrand is -1 on the entire interval 1/2

  • @sword7163
    @sword71634 жыл бұрын

    that was cool

  • @Omcsesz
    @Omcsesz4 жыл бұрын

    At the end, you should substitute x=1 first to the expression 1-2lnx, right?

  • @MichaelPennMath

    @MichaelPennMath

    4 жыл бұрын

    The 1 is outside of the evaluation. Notice that a few steps before we had 1-2integral.

  • @derekcresswell7352
    @derekcresswell73523 жыл бұрын

    I'm a tad confused by one step. Why is turning floor(u) to a constant valid? Wouldn't u take on the value of n + 1 at a single point in the integration?

  • @feitao13

    @feitao13

    Жыл бұрын

    Changing the value of a function at one single point would not affect the integration, i.e., integral of f = integral of g over some interval, if f = g except at one point.

  • @jeremy.N
    @jeremy.N4 жыл бұрын

    lol, I would have done the same thing. except that I would have made at least 3 mistakes with plus and minus. Great video, although your audio is a little much, maybe you can invest into a new micro (a ball like bprp?).

  • @Boe1771

    @Boe1771

    3 жыл бұрын

    Yeah, somehow the audio seems off indeed. But please keep the great maths coming :)

  • @dominiquelaurain6427
    @dominiquelaurain64274 жыл бұрын

    But (-1)^(n+1)/(n+1) equals -1 (not 1) when n = 0 as you say @8:51 ...but parenthesis is then not zero later (you mistakenly substract 1 in parenthesis and add 1 outside). Check out the missing "-" on your formula @9:21 (and remove your trick adding and substracting 1).

  • @rsactuary8763
    @rsactuary87634 жыл бұрын

    I forget the problem every time he turns around and I see his back muscles.

  • @lluisllacer7295
    @lluisllacer72954 жыл бұрын

    How do you know that the sequence $\sum_{n=1}^\infty \int_n^{n+1}(-1)^u {1\over u^2}du$ converges to the same number as $\int_1^\infty (-1)^u{1\over 1}du$ does? I mean, the first one is a subsequence from the second. This doesn't mean that if de subsequence converges the main sequence converges too or, in case both converge, share de same limit.

  • @SlidellRobotics
    @SlidellRobotics3 жыл бұрын

    I broke up into subintervals without the substitution and got the same sum without having to integrate 1/u², just 1 and -1.

  • @jesusandrade1378
    @jesusandrade1378 Жыл бұрын

    Maple Calculator evaluates this definite integral as 1, without the 2 ln(2) term

  • @bookworm8414
    @bookworm84143 жыл бұрын

    The power n of -1 is either 1 or -1, why their sum is not an integer?

  • @abdallahal-dalleh6453
    @abdallahal-dalleh64533 жыл бұрын

    Can you explain 10:00 why we did that?

  • @smiley_1000
    @smiley_10004 жыл бұрын

    Didn't you forget the du when doing the u-substitution?

  • @tokajileo5928
    @tokajileo59284 жыл бұрын

    i lost it at 9:21 when that 2 appeared

  • @jannowak9052
    @jannowak9052 Жыл бұрын

    Czy ty zauważyłeś, że w ostatnich sekundach logo twojego kanału zasłania ostateczny wynik?

  • @WmTyndale
    @WmTyndale4 жыл бұрын

    Nice new integral form. Here is an extension: Do the result (I have not) for [1/x^n] inside the integral. May be interesting.

  • @adamjennifer6437

    @adamjennifer6437

    3 жыл бұрын

    Just watch this impressive Math channel kzread.info/dron/ZDkxpcvd-T1uR65Feuj5Yg.html

  • @romanpavelko5994
    @romanpavelko59944 жыл бұрын

    But inserting x^(n + 1) to the series means that for that series to be convergent x should be -1 < x < 1. How to prove that x is between 0 and 1 only?

  • @user-sk5nl6vj3e

    @user-sk5nl6vj3e

    4 жыл бұрын

    The range of x in the integral is from 0 to 1 so we only consider x in that range.

  • @disgruntledtoons
    @disgruntledtoons3 жыл бұрын

    If the floor is lava, how do we integrate it?

  • @housamkak646
    @housamkak6464 жыл бұрын

    mannnnnnnnnnnnnnnnnnnnn this is amazingggg

  • @AlfonsoNeilJimenezCasallas
    @AlfonsoNeilJimenezCasallas4 жыл бұрын

    Math is crazy!

  • @kkalyan5569
    @kkalyan55694 жыл бұрын

    i have done this as(i.e with 1/x)it is without substitution and ended up with the alternating series of 1-1+1-1+1...... which is of course not the correct one .Someone pls correct me .

  • @Zero-tq6hv

    @Zero-tq6hv

    4 жыл бұрын

    You've probably just made an error during computation. INT from 0 to 1 (-1)^floor(1/x) dx = Sum from n=1 to inf (INT from 1/(n+1) to 1/n (-1)^floor(1/x) dx) = Sum from n=1 to inf (INT from 1/(n+1) to 1/n (-1)^n dx) = Sum from n=1 to inf (((-1)^n)/n+((-1)^(n+1))/(n+1)) After simplification we get 1-2ln(2). Quite simillar to method used in the video.

  • @JonahFoley
    @JonahFoley4 жыл бұрын

    nice problem

  • @fulla1
    @fulla14 жыл бұрын

    Oh, I hate it so much, when people write a summation sign in a hurry (or from an uncomfortable angle) that it looks like the thing at 6:27. But great video, though!

  • @seroujghazarian6343
    @seroujghazarian63433 жыл бұрын

    For the same integral but the roof function instead of floor, the result is the opposite

  • @ethanchandler3934
    @ethanchandler39342 жыл бұрын

    I would write it as 1-ln(4) but very good video’

  • @aranbrico1005
    @aranbrico10054 жыл бұрын

    I got a little bit confused with 2

  • @aranbrico1005

    @aranbrico1005

    4 жыл бұрын

    Never mind.

  • @tubamazouz
    @tubamazouz Жыл бұрын

    👍👍

  • @skwbusaidi
    @skwbusaidi4 жыл бұрын

    In Wolframalpha ∫ (-1)^floor(1/x) dx from 0 to 1 I got 0.50... which not equal to 1-2ln(2)

  • @MichaelPennMath

    @MichaelPennMath

    4 жыл бұрын

    This is one that wolframalpha and Mathematica have a hard time approximating. If you give them some help and change the bound of integration to 0.01 to 1 it works: bit.ly/2JQP5Gm

  • @felipelopes3171

    @felipelopes3171

    4 жыл бұрын

    In general, you should never put an integral with a singularity in a numerical integration software, because it messes up their methods. You can try splitting it and do a change of variables to remove the singularity.

  • @jcfgykjtdk
    @jcfgykjtdk3 жыл бұрын

    I got Ln(4) - 1

  • @DeepDeepEast
    @DeepDeepEast4 жыл бұрын

    Great Content. , I should have watched those when I began my math bachelor. Now I am finished soon and I still lack basic stuff in calculus 😁

  • @pbj4184

    @pbj4184

    3 жыл бұрын

    Wtf? You didn't fail even once?

  • @DeepDeepEast

    @DeepDeepEast

    3 жыл бұрын

    @@pbj4184 No never failed an exam. But also didn't receive good grades sometimes and after I learned for an exam I forgot a lot of things. Or didn't dig deeper into details. The stuff I find interesting I will remember for ever.

  • @pbj4184

    @pbj4184

    3 жыл бұрын

    @@DeepDeepEast Oh that makes sense. Are you working in the industry now?

  • @HideyukiWatanabe
    @HideyukiWatanabe2 жыл бұрын

    7:20 Why can you change the order of sum? The series Σ(-1)^n (1/n) and Σ(-1)^n (1/(n+1)) aren't absolutely convergent.

  • @HideyukiWatanabe

    @HideyukiWatanabe

    2 жыл бұрын

    Oh, it was simple. Adding two convergent series converges to the sum of each series, looking RHS to LHS.

  • @thecustomer2804
    @thecustomer28044 жыл бұрын

    Wait hold on. 1 - 2ln(1 + 0) = 1, so plugging in the limits of integration should get you (1 - 2ln(2)) - (1) = -2ln(2) rightt?

  • @Mr5nan

    @Mr5nan

    4 жыл бұрын

    the bounds are only for the latter term, not for the 1

  • @GhostyOcean

    @GhostyOcean

    4 жыл бұрын

    Some parentheses would've helped, or putting the +1 after the integral. It should be written as 1-(2ln(x+1)|x=1 x=0)

  • @thecustomer2804

    @thecustomer2804

    4 жыл бұрын

    GhostyOcean Mr5nan Thanks for the tips! I make small but obvious mistakes at times

  • @stewartzayat7526
    @stewartzayat75264 жыл бұрын

    Yay! I arrived at the same answer. I wanted to check if I was correct on wolfram alpha, but wolfram got the wrong answer lol

  • @MichaelPennMath

    @MichaelPennMath

    4 жыл бұрын

    wolfram alpha and mathematica have a hard time with this one. If you set the lower bound to 0.01 instead of 0 it will approximate it correctly.

  • @khai-hoannguyen-dang9082
    @khai-hoannguyen-dang90824 жыл бұрын

    Really nice talk

  • @diabl2master
    @diabl2master4 жыл бұрын

    I feel that the substitution was totally unecessary. I arrived at the sum on the right at 7:37 doing essentially exactly the same thing but without the substitution.

  • @xshortguy

    @xshortguy

    4 жыл бұрын

    Well, in all honesty a substitution is never really necessary, it just makes things easier to see.

  • @MichaelJamesActually
    @MichaelJamesActually Жыл бұрын

    feel like I need a cigarette after that.

  • @albertreitsma988
    @albertreitsma9882 жыл бұрын

    Missed the usual "that's a good place to stop"

  • @matthewryan4844

    @matthewryan4844

    2 жыл бұрын

    Good place to stop: ∅

  • @muhammadsarimmehdi
    @muhammadsarimmehdi4 жыл бұрын

    Why are you adding x around 10:12?

  • @pbj4184

    @pbj4184

    3 жыл бұрын

    To get an integral and then switch the order of summation and integration to solve the problem

  • @pbj4184

    @pbj4184

    3 жыл бұрын

    Michael does that a lot

  • @VUrosov
    @VUrosov3 жыл бұрын

    It's dx=-du/u^2

  • @tomhase7007
    @tomhase70073 жыл бұрын

    Reordering a conditionally convergent series? Mh...

  • @mashtonish
    @mashtonish4 жыл бұрын

    I got immediately lost at 10:00

  • @ByteOfCake

    @ByteOfCake

    3 жыл бұрын

    hes essentially just multiplying it by 1. If you use the bound he used you get 1^(n+1)-0^(n+1) which is just 1.

  • @samiulfahim5384
    @samiulfahim53843 жыл бұрын

    What a body 👍 , Sirrrrrr 😉 .

  • @derletsplayer9140
    @derletsplayer91404 жыл бұрын

    nice video but why does he scream?

  • @paulkohl9267
    @paulkohl92674 жыл бұрын

    The answer is approximately -0.338629436111989061883446424291635, which I mean, come on,, obviously! ;)